<<

27 Drug John A. Renner, Jr., MD, and E. Nalan Ward, MD

KEY POINTS The National Survey on Drug Use and Health (NSDUH) findings in 2011 showed that 22.5 million Americans, or 8.7% Incidence of the population ≥ age 12, used an illicit drug in the past month.2 Marijuana continues to be the most commonly- • Depending on the drug used and the clinical setting, abused illicit drug, followed by non-medical use of prescribed up to 50% of patients in mental-health treatment will or over-the-counter (OTC) medication abuse (Figure 27-1). have a co-occurring substance-use disorder. During the last decade, the number of patients treated for Epidemiology –related problems in the US has grown stead- ily. Between 2004 and 2011, the number of emergency depart- • Substance abuse is a major public health problem ment (ED) visits for drug-related events increased by 100%.3 that affects a large number of psychiatric patients. In the same period, the number of drug-related suicide • Screening for substance use and abuse should be a attempts rose by 41%. Adolescents and young adults were the routine part of all mental-health evaluations. most vulnerable to the adverse effects of drug use. The major- • The problem is particularly severe in public sector ity of ED visits for those in these age groups were the result treatment settings. of medical emergencies related to drug misuse/abuse (Figure 27-2). Prognosis In 2011, and marijuana were the most commonly- • Research has demonstrated that integrated treatment used illicit drugs that led to ED visits.3 Non-medical use of delivered in settings that are skilled in the prescription drug or OTC medication–related ED visits management of both mental-health and substance- increased by 132% between 2004 and 2011, with opiate/ use disorders will significantly improve outcome. involvement rising by 183%.3 Treatment Options These results are not surprising in that they reflect the • Brief interventions, motivational-enhancement extent of the prescription pain-reliever addiction epidemic in the US. About 4.5 million individuals reported current non- therapy, cognitive-behavioral therapy, and 2 pharmacotherapy with methadone and medical use of pain relievers in 2011. buprenorphine are each efficacious for . In recent years, the liberal prescription of potent opioid medications, as well as their increased availability and diver- • Psychiatrists need to become adept in the use of sion in communities, have played a significant role in the evidence-based treatment for substance-use disorders. development of this public health problem.4 More than half • The availability of effective psychotherapies and of those who use illicit pain relievers reported that they pharmacotherapies for addictive disorders makes it had obtained the pain relievers from family or friends2 possible to successfully manage patients (in (Figure 27-3). outpatient settings) who are addicted to opiates and cocaine. Complications • Unrecognized and untreated substance-use disorders are associated with poor outcomes and treatment Illicit drugs 22.5 failure for co-occurring mental health disorders. Marijuana 18.1

Psychotherapeutics 6.1

Cocaine 1.4 OVERVIEW The chronic, relapsing nature of substance abuse is inappro- 1.0 priately thought to imply that substance-abuse treatment is not helpful. This leads clinicians to ignore multiple opportu- 0.6 nities to intervene in the disease process. Most clinicians fail to appreciate that the relapse rate of other common chronic medical disorders (e.g., diabetes, hypertension, asthma) Heroin 0.3 exceeds that for substance-use disorders.1 Problems related to substance abuse should always be addressed with the same 0 5 10 15 20 25 degree of compassion and persistence that is directed to other common relapsing medical disorders. Numbers in millions Among individuals who abuse drugs, 53% have a co- Figure 27-1. Past month illicit drug use among persons aged 12 or occurring psychiatric disorder. Successful treatment of this older: 2011. (Findings from National Survey on Drug Use and Health, expanding group of patients requires that clinicians improve 2011: Summary of National Findings, NSDUH Series H-44, HHS their skills in the management of patients with substance-use Publication No. (SMA) 12-4713. Rockville, MD: Substance Abuse and disorders and co-occurring psychiatric disorders. Mental Health Services Administration, 2012.)

291

Downloaded for Rohul Amin ([email protected]) at Uniformed Services Univ of the Health Sciences from ClinicalKey.com by Elsevier on September 29, 2018. For personal use only. No other uses without permission. Copyright ©2018. Elsevier Inc. All rights reserved. 292 PART IX Psychiatric Disorders

3,000 Visits for misuse/abuse THE NEUROBIOLOGY OF ADDICTION Visits for other than misuse/abuse Disruption of the endogenous reward systems in the brain is a common feature of drug abuse; most addictive drugs act by 2,500 disrupting central nervous system (CNS) dopamine circuits. Acutely, synaptic dopamine increases and circuits that mediate motivation and drive, conditioned learning, and inhibitory 2,000 controls are disrupted (Figure 27-4). This enhancement of synaptic dopamine is particularly rewarding for individuals with abnormally low density of the D dopamine receptor 1,500 2 (D2DR). Normal individuals (with normal D2DR levels) find this experience too intense and aversive and thus may be shielded from the risk of addiction. Low D DR availability is 1,000 2 associated with an increased risk for abuse of cocaine, heroin, , , and . Chronic 500 drug use produces long-lasting and significant decreases in dopamine brain function, manifested by decreases in both the Rate of ED vistits per 100,000 population D2DR and dopamine cell activity. These decreases are also 0 associated with dysfunction in the prefrontal cortex, including the orbitofrontal cortex (which is involved in salience attribu- 65 + 0–5 tion) and the cingulated gyrus (which is involved in inhibitory 6–11 21–24 12–17 18–20 25–29 45 –54 66 –64 30 –34 35 – 44 control and mood regulation). Low baseline levels of beta- Figure 27-2. Rates of drug-related ED visits per 100,000 population, plasma endorphins are associated with a higher endorphin by age group, 2011. (From Substance Abuse and Mental Health response to alcohol and an increased risk for alcohol depend- Services Administration, Center for Behavioral Health Statistics and ence; it is less clear whether this abnormality also increases Quality. Drug Abuse Warning Network, 2008: National Estimates of the risk for opiate dependence. Table 27-1 lists the major drugs Drug-Related Emergency Department Visits. HHS Publication No. of abuse and the associated disruption of CNS neurotransmit- SMA 11-4618. Rockville, MD, 2011, HHS.) ter systems.

COCAINE Abuse The percent of persons with or abuse decreased between 2006 and 2011 from 0.7 to 0.3 %.2 In 2011,

SOURCE WHERE USER OBTAINED More than one doctor (1.9%) Free from friend/relative (54.2%) One doctor (18.1%)

SOURCE WHERE FRIEND/RELATIVE OBTAINED

One doctor (81.6%) Other (5.0%) Bought on internet (0.3%)

Drug dealer/ stranger (3.9%)

More than one doctor (3.1%) Free from friend/ Bought/took from relative (5.5%) friend/relative (16.6%) Bought/took from friend/relative (5.7%) Drug dealer/ Other (2.2%) stranger (5.7%)

Bought on Internet (0.2%) Figure 27-3. Source where pain relievers were obtained for most recent non-medical use among past year users aged 12 or older: 2010–2011. The Other category includes the sources “Wrote Fake Prescription,” “Stole from Doctor’s Office/Clinic/Hospital/Pharmacy,” and “Some Other Way.” (Findings from National Survey on Drug Use and Health: Summary of National Findings, NSDUH Series H-44, HHS Publication No. (SMA) 12-4713. Rockville, MD: Substance Abuse and Mental Health Services Administration, 2012.)

Downloaded for Rohul Amin ([email protected]) at Uniformed Services Univ of the Health Sciences from ClinicalKey.com by Elsevier on September 29, 2018. For personal use only. No other uses without permission. Copyright ©2018. Elsevier Inc. All rights reserved. Drug Addiction 293

Nicotine Alcohol 27 +

Opiates Opioid Glutamate inputs peptides (e.g., from cortex) – Opiates Alcohol ? GABA – – VTA interneuron PCP Alcohol + ? DA DA + + – Cannabinoids Glutamate inputs (e.g., from amygdala PPT/LDT) VTA NAc

Figure 27-4. Converging acute actions of drugs of abuse on the ventral tegmental area and nucleus accumbens. DA, dopamine; GABA, γ-aminobutyric acid; LDT, laterodorsal tegmentum; NAc, nucleus accumbens; PCP, phencyclidine; PPT, pedunculopontine tegmentum; VTA, ventral tegmental area. (Redrawn from Nestler EJ. Is there a common molecular pathway for addiction? Nat Neurosci 8(11):1445–1449, 2005.)

TABLE 27-1 Neurobiology of Drug cocaethylene, an active metabolite that has a longer half-life (2–4-hours) and is more cardiotoxic than cocaine. The com- Drug Type Mechanism of Reinforcement bination of cocaine and marijuana also produces more intense Cocaine Mesolimbic dopamine system , higher plasma levels, and more cardiotoxicity than does cocaine alone. Nicotine The signs and the symptoms of acute Mesolimbic dopamine system are similar to those of abuse. Typical com- Alcohol GABA and glutamate plaints associated with intoxication include anorexia, insom- Dopamine and serotonin Opioid peptide systems nia, anxiety, hyperactivity, and rapid speech and thought Cannabinoids Dopamine in the nucleus accumbens processes (“speeding”). Signs of adrenergic hyperactivity (such as hyperreflexia, tachycardia, diaphoresis, and dilated pupils GABA, γ-aminobutyric acid. responsive to light) may also be seen. More severe symptoms (e.g., hyperpyrexia, hypertension, cocaine-induced vasospastic events [e.g., stroke or myocardial infarction]) are relatively rare the number of persons with cocaine dependence or abuse was among users, but are fairly common in those seen in hospital roughly 821,000. Despite the downward trend, cocaine, after EDs. Patients may also manifest stereotyped movements of the alcohol, remained the leading substance of abuse related to mouth, face, or extremities. Snorting the drug may produce frequent ED contacts, general hospital admissions, family vio- rhinitis or sinusitis and, rarely, perforations of the nasal lence, and other social problems.3 Cocaine use resulted in septum. Free-basing (inhalation of cocaine alkaloid vapors) 40% of all illicit drug-related ED visits in 2011. Even individu- may produce bronchitis. Grand mal are another infre- als with normal psychological profiles are vulnerable to com- quent complication. Patients also describe “snowlights” (i.e., pulsive cocaine use. Acute use leads to intense euphoria that flashes of light usually seen at the periphery of the visual is often associated with increased sexual desire and with field). Crack is a highly addictive free-base form of cocaine improved sexual function. These rewards are often followed that is sold in crystals and can be smoked. by a moderate-to-severe post–cocaine use depression that The most serious psychiatric problem associated with stimulates a strong incentive for further cocaine use. These chronic cocaine use is a cocaine-induced psychosis (manifest responses are primarily mediated by disruptions of synaptic by visual and auditory hallucinations and paranoid delusions dopamine. The initial cocaine response is a function of ele- often associated with violent behavior). Tactile hallucinations vated dopamine generated by blockade of the dopamine (called “coke bugs”) involve the perception that something is reuptake transporter (DAT) and the inhibition of the reuptake crawling under the skin. A cocaine psychosis may be indistin- of synaptic dopamine. Chronic cocaine use leads to down- guishable from an amphetamine psychosis, but it usually does regulation of dopamine receptors and ultimately to depletion not last as long. High doses of stimulants can also cause a state of synaptic dopamine, which is thought to be the cause of of excitation and mental confusion known as “ post–cocaine use depression (Figure 27-5). Like other stimu- delirium.” lants, cocaine also disrupts the synthesis and reuptake of sero- tonin. Other receptors affected include norepinephrine, Management N-methyl-D-aspartate (NMDA), gamma-aminobutyric acid (GABA), and opioid receptors. Plasma cholinesterases rapidly Cocaine abuse became common among affluent young people convert cocaine into benzoylecognine (BE), an inactive metab- in the early to mid-1980s, but with the availability of packaged olite that can be detected in the urine for 3 days. When alcohol smokable cocaine, or crack, in low-cost doses, all classes and is taken in conjunction with cocaine, liver esterases produce racial groups have become potential users. Occasional cocaine

Downloaded for Rohul Amin ([email protected]) at Uniformed Services Univ of the Health Sciences from ClinicalKey.com by Elsevier on September 29, 2018. For personal use only. No other uses without permission. Copyright ©2018. Elsevier Inc. All rights reserved. 294 PART IX Psychiatric Disorders

Ventral Nucleus tegmental accumbens Cocaine Activated Adenylyl area proteins cyclase

Protein kinase A ATP cAMP Dopaminergic Uptake Gs neuron transporter D1-like receptor

Tyrosine Dopamine

D2-like receptor Presynaptic G GABA receptor i receptor cAMP

Opioid ATP receptor Opioid receptor Gi cAMP GABA ATP GABAergic Heroin Heroin interneuron

Figure 27-5. Schematic of the effects of cocaine and heroin in the synapse. (Redrawn and adapted from Leshner A and the US National Institute on Drug Abuse. New understandings of drug addiction, Hospital Practice Special Report, 1997. New York, 1997, McGraw-Hill.) use does not require specific treatment except in the case of a For the cocaine addict, the compulsion to use is over- life-threatening overdose. Most potentially lethal doses are whelming. For this reason, a hospitalized, cocaine-dependent metabolized within 1 hour. In the interim, intubation and patient should be monitored closely and should have a drug assisted breathing with oxygen may be necessary. Stroke has screen performed after behavioral change, particularly after been reported, and death can be caused by ventricular fibril- departures from the floor or receiving visitors. Urine should lation or myocardial infarction. The cardiac status should be examined for cocaine metabolites and, preferably, for all therefore be monitored closely. High doses of drugs of abuse. are recommended for management of stimulant-induced Once compulsive cocaine use has begun, it is almost delirium and agitation. Neuroleptics should be avoided impossible for the user to return to a pattern of occasional, because of the risk of potentially fatal hyperthermia. Intrave- controlled use. Such individuals are also likely to develop nous (IV) diazepam should be used to control convulsions. problems with alcohol and other drugs. For that reason, the Chronic cocaine use produces tolerance, severe psychologi- goal of treatment should be abstinence from cocaine and all cal dependency, and physiological dependence (marked by other drugs. All cocaine abusers should be referred for indi- irritability, anhedonia, low mood, insomnia or hypersomnia, vidual or group counseling, and participation in 12-step and anxiety).5 Dependent users typically follow a cyclical self-help programs should be strongly recommended. Manual- pattern of 2 or 3 days of heavy binge use, followed by a with- guided cognitive-behavioral therapy (CBT) has been effica- drawal “crash.” Use is resumed again in 3 to 4 days, depending cious in the treatment of cocaine dependence.7 Twelve-step on the availability of cash and the drug. A gradual reduction facilitation and CBT appear to be helpful, particularly in indi- in use of the drug is almost never possible. Detoxification is viduals with more severe dependence and in those with accomplished by the abrupt cessation of all cocaine use, co-morbid disorders.8 Family members or significant others usually through restricted access (e.g., a loss of funds or con- should be referred separately to Al-Anon because they will tacts, or incarceration). Symptoms of withdrawal begin to gain insights that may help them eliminate systemic support resolve within 7 days; the value of medication treatment for for the patient’s drug use. There is no Food and symptoms has yet to be confirmed. Drugs that Administration (FDA)–approved pharmacotherapy for cocaine enhance CNS catecholamine function may reduce craving, dependence. Trials with desipramine, fluoxetine, bupropion, although they are of limited clinical benefit and they have not amantadine, and carbamazepine have had inconsistent results. been proven effective in double-blind placebo-controlled Positive responses have been reported in trials with topiram- trials. There is some indication that amantadine (an indirect ate, baclofen, and modafinil, but these drugs require further dopamine agonist) and propranolol may be helpful to indi- investigation. Several trials with disulfiram have shown benefit, viduals with severe withdrawal symptoms. The major compli- with reduced craving and use, and a reported increase in cation of withdrawal is a severe depression with suicidal the aversive effects of cocaine should the patient relapse. ideation.6 If this occurs, the patient typically requires psychi- These reactions are thought to be mediated by the inhibitory atric hospitalization. The need for inpatient care may be time- effect of disulfiram on dopamine beta-hydroxylase. This limited, since suicidal ideation usually clears promptly with action will elevate depleted plasma dopamine levels in chronic the cessation of cocaine use. A less severe anhedonic state may users and will produce abnormally high dopamine levels if persist for 2 to 3 months and is thought to reflect a more cocaine is ingested; this results in a dysphoric experience for persistent state of dopamine depletion. most. users

Downloaded for Rohul Amin ([email protected]) at Uniformed Services Univ of the Health Sciences from ClinicalKey.com by Elsevier on September 29, 2018. For personal use only. No other uses without permission. Copyright ©2018. Elsevier Inc. All rights reserved. Drug Addiction 295

AMPHETAMINES users and abusers. Illicit amphetamine and methampheta- mine (speed) use accounted for 12.8 % of all illicit-drug-use- 27 Abuse related ED visits in 2011.3 In 2011, roughly 970,000 persons ≥ 12 years were active non- The signs and symptoms of acute amphetamine intoxica- medical users of prescription stimulants.2 Between 2004 and tion are similar to those of cocaine abuse. Long-term effects 2011, prescription CNS stimulants led to a striking 307% include depression, brain dysfunction, and weight loss. increase in ED visits. Among these agents, the ADHD drug In addition, either with acute or chronic amphetamine amphetamine-dextroamphetamine (e.g., Adderall®) showed a intoxication a paranoid psychosis without delirium can 650% increase during that period.3 develop. Although typically seen in young people who use IV Illicitly-produced methamphetamine fueled an epidemic methamphetamine hydrochloride, paranoia can also occur in of abuse on the West Coast and in much of the Midwest in chronic users of dextroamphetamine or other amphetamines. the 1990s. Since then, the number of methamphetamine A paranoid psychosis may also occur with or without other abusers had declined as a result of stricter federal controls on manifestations of amphetamine intoxication. The absence of the production and distribution of certain medications. disorientation distinguishes this condition from most other The primary action of these drugs is an increase in synaptic toxic psychoses. This syndrome is clinically indistinguishable dopamine via the release of dopamine into the synapse; meth- from an acute schizophrenic episode of the paranoid type, and amphetamine also blocks the DAT. This produces a dopamine the correct diagnosis is often made in retrospect, based on a “high” that is both more intense and longer lasting than history of amphetamine use and a urine test that is positive results from cocaine, lasting anywhere from 8 to 24 hours. for amphetamines. Use of haloperidol or low-dose atypical Methamphetamine, invented for military use by the Japanese antipsychotics is often effective in the acute management of in World War I, is currently a schedule II drug, that can be this type of substance-induced psychosis. taken orally (“speed”), taken anally, smoked (“crystal”), Other distinctive features of chronic stimulant abuse snorted, or injected. It has been approved for the treatment of include dental problems (e.g., caries, missing teeth, bleeding ADHD and obesity. Long-term use of amphetamines can cause and infected gums), muscle cramps (related to dehydration cognitive impairment (including dulled awareness, decreased andw lo levels of magnesium and potassium), constipation intellectual capacity, memory impairment, and motor retarda- (due to dehydration), nasal perforations, and excoriated skin tion). Positive positron emission tomography (PET) scans lesions (speed bumps) (Figure 27-6). The urine may have a show loss of DAT in the caudate and the putamen, and mag- stale smell due to ammonia constituents used in the illicit netic resonance imaging (MRI) studies show decreased per- manufacture of methamphetamine. fusion in the putaman and the frontal cortex as well as loss of volume in both the amygdala and the hippocampus. Routine medical evaluation may uncover the most common type of Treatment amphetamine abuse seen in clinical settings (involving use of Amphetamines can be withdrawn abruptly. If the intoxication amphetamines to control obesity and that later led to chronic is mild, the patient’s agitation can be handled by reassurance amphetamine abuse). Amphetamine abusers quickly develop alone. The patient can be “talked down,” much as one might tolerance and may use 100 mg each day in an unsuccessful handle an adverse D-lysergic acid diethylamide (LSD) reaction. effort to control weight. This type of amphetamine abuse can If sedation is necessary, benzodiazepines are the drugs of be treated by abruptly discontinuing the drug or by gradually choice. should be avoided because they may tapering the dose. In either case, the patient should be given heighten dysphoria and increase the patient’s agitation. Hyper- a more appropriate program for weight control. tension will usually respond to sedation with benzodiazepines. A more serious problem involves the patient who develops When severe hypertension arises, phentolamine is recom- a severe on amphetamines and mended for vasodilation. Beta- or mixed alpha- and beta- whoy ma have the same symptoms seen in younger street-drug adrenergic blockers (such as propranolol or labetalol) are to

A B

Figure 27-6. Face of a patient with chronic methamphetamine abuse. Before use (A) and after 3 months of use (B). (From the Faces of Meth educational program. Copyright 2005 Multnomah County [Oregon] Sheriff’s Office, Portland, Oregon.)

Downloaded for Rohul Amin ([email protected]) at Uniformed Services Univ of the Health Sciences from ClinicalKey.com by Elsevier on September 29, 2018. For personal use only. No other uses without permission. Copyright ©2018. Elsevier Inc. All rights reserved. 296 PART IX Psychiatric Disorders be avoided because they may exacerbate stimulant-induced narcolepsy. GHB has a relatively low therapeutic index; as little cardiovascular toxicity. as twice the dose that produces euphoria can cause CNS Most signs of intoxication clear in 2 to 4 days. The major depression. In overdose it can produce a potentially fatal problem is management of depression upon discontinuation coma; it has also been identified as a “date rape” drug. Keta- of amphetamine use. In mild cases, this depression can be mine (“Special K,” “Super K,” or “K”) is a non-competitive manifesty b lethargy, as well as by the temptation to use NMDA antagonist that is classified as a dissociative anesthetic. amphetamines for energy. In more serious cases, the patient It is currently used as a veterinary anesthetic and it can produce may become suicidal and require inpatient psychiatric treat- delirium, amnesia, and respiratory depression when abused. ment. The efficacy of antidepressants in such cases has not Ketamine, like phencyclidine (PCP), binds to the NMDA been adequately documented. Even with support and psycho- receptor site and blocks the action of excitatory neurotrans- therapy, most patients experience symptoms of depression for mission; it affects perceptions, memory, and cognition. More 3 to 6 months following the cessation of chronic ampheta- recently, studies suggest it can rapidly reverse treatment- mine abuse. CBT has been helpful, but it may need to be refractory depression. adapted to allow for the cognitive impairment associated with The treatment for overdoses of all of these drugs is prima- long-term methamphetamine use. rily symptomatic.

CLUB DRUGS OPIOIDS During the 1990s the abuse of “club drugs,” primarily While abuse of all major drug classes increased during the 3,4-methylenedioxy-methamphetamine (MDMA, or “ecstasy”), 1990s, the most dramatic increase was seen among new γ-hydroxybutyrate (GHB), and ketamine steadily increased. abusers of prescription pain relievers. In 2011, an estimated This trend was reversed between 1998 and 2001 when the 4.5 million (1.7%) individuals were active non-medical users Monitoring the Future Survey (MFS) reported a steep decline of pain relievers.2 In the US, prescription pain reliever-abuse in the use of ecstasy. The drop has been attributed to a general is considered an epidemic. Many public health problems have recognition of the dangers associated with the use of this drug. been associated with this particular type of drug abuse. The In 2012, MFS reported a 1.5% past-year use of club drugs perception of prescription drugs as being less harmful than among 12th graders.9 illicit drugs likely contributed to the problem.2 MDMA has both amphetamine-like and hallucinogenic Due to frequency of prescription pain relievers, they are effects. Its primary mechanism of action is via indirect considered an “entry drug” after illicit marijuana. According serotonin agonism, but it also affects dopamine and other to the 2012 MFS, 1 out of every 12 high school seniors reported neurotransmitter systems. These club drugs increase synaptic taking prescription pain relievers for non-medical use within dopamine and alter serotonergic neurotransmission. MDMA the last year.9 Unfortunately, with the progression of the was initially used experimentally to facilitate psychotherapy, abuse, many individuals turn to heroin. The latest NSDUH but its use was banned after it was found to be neurotoxic to data estimated a significant increase in heroin use in 2007 animals. The intense feelings of empathy experienced by users (373,000) to 2011 (620,000).2 The major health concerns with may be a result of the flooding of the serotonin system. In increased heroin use are intravenous (IV)-related medical toxic amounts, it produces distorted perceptions, confusion, complications. hypertension, hyperactivity, and potentially fatal hyperther- Nearly one-third (31%) of individuals with acquired mia. With chronic use, serotonin stores are depleted and sub- immunodeficiency syndrome (AIDS) in the US are related to sequent doses produce a less robust high and more unpleasant injection drug use.10 An estimated 70% to 80% of the new side effects (such as teeth gnashing and restlessness). Frequent hepatitis C infections occurring in the US each year are among users learn to anticipate these effects and tend to limit their IV drug users. Other public health problems that have emerged long-term consumption of the drug. over the last decade include increased ED visits and deaths due GHB (sodium oxybate) is structurally similar to GABA and to overdoses. Specifically, prescription methadone, oxycodone it acts as a CNS depressant. It has been approved by the FDA and hydrocodone-related ED visits quadrupled between 2004 as a schedule III controlled substance for the treatment of and 200811 (Figure 27-7).

140,000

Methadone 127,487 120,000 Oxycodone Hydrocodone 93,824 104,335 100,000

76,919 80,000 62,469 73,945 70,373 60,000 51,417 66,114 55,704 60,418 46,535 49,442 40,000 46,466 40,899

20,000

0 2004 2005 2006 2007 2008 Figure 27-7. Emergency department visits related to methadone,oxycodone and hydrocodone: 2004–2008. DAWN 2008.

Downloaded for Rohul Amin ([email protected]) at Uniformed Services Univ of the Health Sciences from ClinicalKey.com by Elsevier on September 29, 2018. For personal use only. No other uses without permission. Copyright ©2018. Elsevier Inc. All rights reserved. Drug Addiction 297

Death rates Caudate Putamen Corpus Deaths per 100,000 population nucleus callosum 27 (head) 25 Caudate nucleus (tail) 20 Motor vehicle accidents

15 Ventral tegmental area 10 All Locus poisonings Prefrontal Legal and cortex ceruleus 5 illegal drug Nucleus Fourth poisonings accumbens ventricle Pons 1980 1985 1990 1995 2000 2005 2008 Temporal lobe Cerebellum Medial Drug deaths forebrain In thousands bundle Substantia 16 Non-opioid nigra drugs Figure 27-10. Schematic of reward pathways in the brain. (Redrawn 12 and adapted from Leshner A and the US National Institute on Drug Abuse. New understandings of drug addiction, Hospital Practice 8 Opioid Special Report, April 1997. New York, 1997, McGraw-Hill.) painkillers* Unspecified 4 drugs TABLE 27-2 Opioid Agonist Drug Effects Acute-use effects Euphoria 2000 2005 2008 Vomiting *Note: morphine, hydrocodone, oxycodone, methadone, fentanyl and others Constricted pupils Figure 27-8. Overdose death rates. (Warner M, Chen LH, Makuc DM, Depressed respiration Drowsiness Anderson RN, Miniño AM. Drug poisoning deaths in the United States, Decreased pain sensation 1980–2008. NCHS data brief, no 81. Hyattsville, MD, 2011, National Decreased awareness Center for Health Statistics.) Decreased consciousness Large-dose acute-use Non-responsiveness effects Pin-point pupils 100 If severe anoxia, pupils may dilate 90 Full agonist Bradycardia and hypotension (e.g., heroin) Skin cyanotic 80 Skeletal muscle flaccid Pulmonary edema in approximately 50% 70 Slow or absent respiration 60 Chronic-use effects Partial agonist Psychological dependence 50 (e.g., buprenorphine) Lethargy and indifference 40 Reduction in bowel movement 30 20

% Mu receptor intrinsic activity sedation, decreased secretions, nausea, vomiting, constipa- 10 tion, miosis, urinary hesitation, and hypotension [Table Antagonist (e.g., naloxone) 27-2]). The classic signs of opiate withdrawal are easily recog- 0 nized and usually begin 8 to 12 hours after the last dose (of No drug Low dose High dose a short-lasting agent) (Box 27-1). The patient generally admits Drug dose the need for drugs and shows sweating, yawning, lacrimation, Figure 27-9. Comparison of activity levels of opiates at the mu tremor, rhinorrhea, marked irritability, dilated pupils, pilo- receptor. erection (“gooseflesh”), and an increased respiratory rate. More severe signs of withdrawal occur 24 to 36 hours after the last dose and include tachycardia, hypertension, insomnia, In 2008, there were more poisoning-related deaths than nausea, vomiting, and abdominal cramps. Untreated, the syn- deaths caused by motor vehicle accidents (Figure 27-8). In drome subsides in 3 to 7 days. Withdrawal symptoms are 2011, pain relievers accounted for 46% of all medical emergen- similar in patients addicted to methadone, but they may not cies associated with non-medical use of pharmaceuticals. appear until 24 to 36 hours after the last dose (because of Since 2007, there were more overdose deaths with prescription methadone’s longer half-life) and abate over 2 to 4 weeks. pain relievers than there were with heroin and cocaine Patients addicted to oxycodone may present with a particu- combined.12 larly severe and prolonged withdrawal syndrome and may Opiates act by binding to the mu opioid receptor (Figure require high doses of opiates for adequate control. 27-9). Binding to receptors in the ventral tegmental area stim- As the treatment of opioid dependence becomes more ulates the release of dopamine (Figure 27-10), which activates commonplace on medical and surgical floors of general hos- brain reward centers in the nucleus accumbens. Opiates pitals, physicians are challenged to provide proper manage- produce a wide range of effects (including analgesia, euphoria, ment, necessitating up-to-date knowledge of FDA regulations

Downloaded for Rohul Amin ([email protected]) at Uniformed Services Univ of the Health Sciences from ClinicalKey.com by Elsevier on September 29, 2018. For personal use only. No other uses without permission. Copyright ©2018. Elsevier Inc. All rights reserved. 298 PART IX Psychiatric Disorders

documentation of a 1-year history of addiction. When sublin- BOX 27-1 Signs and Symptoms of Opiate Withdrawal gual (SL) buprenorphine is dispensed in a combination tablet (with naloxone), it has minimal potential for IV abuse • Dysphoric mood and has been effective for maintenance treatment.14 Buprenor- • Nausea vomiting ± phine has been approved for use in the office-based treatment • Body aches of opiate dependence and it provides an attractive alternative • Lacrimation to methadone treatment for higher-functioning individuals • Rhinorrhea and for those with shorter histories of opiate dependence. To • Pupillary dilation initiate buprenorphine treatment, a patient should be • Sweating instructed to refrain from the use of heroin, or any other • Piloerection opiate, for at least 24 hours. Once opiate withdrawal is docu- • Diarrhea mented (and monitored with an opiate withdrawal scale, such • Yawning as the Clinical Opiate Withdrawal Scale [COWS]), treatment • Mild fever should begin with 4 mg/1 mg of SL buprenorphine/naloxone. • Insomnia The patient should be observed for 1–4 hours after the initial • Irritability dose for any signs of precipitated withdrawal. Additional • Opioid craving doses of 4 mg/1 mg can be given every 2–4 hours as needed to stabilize the patient. Most clinicians do not prescribe more than 8–12 mg/2–3 mg on the first day. Should precipitated and community treatment resources, as well as competence in withdrawal occur, more aggressive dosing is recommended to the management of detoxification and opiate substitution manage the withdrawal symptoms. Most patients can be therapy. maintained on SL doses in the range of 12 to 16 mg/day; an adequate stabilizing dose can usually be achieved within 2 to 3 days. Opiate Substitution Therapy If a patient on is to be switched FDA regulations define opiate substitution therapy (with to buprenorphine, the methadone dose should be gradually either methadone, levo-alpha-acetylmethadol [LAAM], or reduced to 30 mg per day. The patient should be maintained buprenorphine) as treatment with an approved opiate that on that dose for 1–2 weeks before being transferred to extends beyond 30 days. An addicted individual cannot be buprenorphine. To avoid precipitated withdrawal, the first placed into a methadone maintenance treatment unless he or buprenorphine dose should not be administered until a mild she manifests physiological evidence of current addiction level of opiate withdrawal is evident (in the range of 5 to 13 (withdrawal signs) and can document a 1-year history of on the COWS). Most patients will usually need to wait 24 to addiction. The only exceptions to this rule are being pregnant 48 hours after their last methadone dose before buprenor- and addicted; being addicted and hospitalized for the treat- phine can be safely administered. The dosing procedure is ment of a medical, surgical, or obstetric condition; and having similar to that described previously for patients using heroin. been addicted and recently released from incarceration. In the Long-term methadone patients usually require higher methadone clinic setting, initiation of maintenance treatment buprenorphine doses (in the range of 16 to 24 mg for stabi- begins with an oral dose of 20 to 30 mg per day. Increases are lization). Because of the ceiling effect seen with partial opiate made daily in increments of 10 mg until a dose is achieved agonists, there is no pharmacological benefit from doses that eliminates withdrawal symptoms and blocks craving. higher than 32 mg/day. Extensive research has shown that More rapid dose escalation runs the risk of excessive sedation opiate substitution therapy is highly effective. It reduces illicit and prolongation of the QTc interval. Doses in the range of drug, use the mortality rate, criminal behavior, and transmis- 800 to 12 mg per day are required to stabilize most addicts sion of hepatitis and human immunodeficiency virus (HIV) and to block the euphoric effect of illicit opiates. Success in infection, and permits many of those with addictions to attain 15 methadone maintenance treatment has been associated with normal levels of social function. Evaluations conducted higher doses (range 60 to 120 mg daily), long-term treatment, under the Drug Addiction Treatment Act of 2000 demon- and the provision of comprehensive counseling and rehabili- strated a very positive response to the introduction of tation services. buprenorphine treatment for opiate dependence. Of the more In addition to methadone, FDA-approved medications for than 104,600 individuals that had been treated, approximately opiate substitution therapy include LAAM (a synthetic opiate 35% had never been in treatment before and 60% were new 16 with a duration of action of 48 to 72 hours) and buprenor- to treatment with medication (Figure 27-11). Compared to phine (a long-acting partial opiate agonist). Patients on LAAM patients on methadone, patients attracted to buprenorphine must be monitored for evidence of a prolonged QTc interval. treatment are more likely to be white, female, better educated, Buprenorphine, a partial opioid agonist, produces a milder and employed (Figure 27-12). state of opiate dependence. Because it only partially activates opiate receptors, buprenorphine does not suppress brainstem Opiate Antagonist Therapy function and it is relatively safe in overdose (Figure 27-8). Buprenorphine has a high affinity for opiate receptors and is Naltrexone is an opioid receptor antagonist which has been slowly dissociated from the receptor. It will displace most available as an oral agent for many years. It fully occupies the other opiates from the receptor and may precipitate opiate opioid receptor and prevents the euphoric effects of in dependent individuals if other opiates are agonists such as heroin, oxycodone etc. Therefore, patients present.o T avoid this problem, the initial buprenorphine dose taking naltrexone would not get the reinforcing “high” feeling should not be administered until the patient demonstrates and that would result in decrease in use pattern. Medication mild-to-moderate symptoms of withdrawal. should be given 7–10 days after last opioid use to prevent Buprenorphine can be used in the treatment of patients precipitated withdrawal symptoms. Oral naltrexone has been who meet Diagnostic and Statistical Manual of Mental Disorders, documented to have better outcomes among motivated heath Fifth Edition (DSM-5), criteria for opioid dependence.13 Unlike care professionals and within the criminal justice system methadone, treatment with buprenorphine does not require population.

Downloaded for Rohul Amin ([email protected]) at Uniformed Services Univ of the Health Sciences from ClinicalKey.com by Elsevier on September 29, 2018. For personal use only. No other uses without permission. Copyright ©2018. Elsevier Inc. All rights reserved. Drug Addiction 299

from the presenting illness and been referred to an opiate 100% Addiction physician survey substitution treatment program. 27 80% Longitudinal patient study Patients on long-term methadone therapy should continue 60% toe receiv daily oral methadone treatment while hospitalized. If a switch to parenteral medication is necessary, methadone 40% can be given in IM doses of 5 or 10 mg every 8 hours. This 20% regimen should keep the patient comfortable regardless of the Patients treated previous oral dose. An alternative method is to give one-third 0% of the daily oral dose intramuscularly every 12 hours. As soon New to New to Transitioned Addicted to as oral medication can be tolerated, the original oral dose substance medication- from non-heroin should be re-instated.17 abuse assisted methadone opioids* Establishing the appropriate dose of methadone for a street treatment treatment addict is a trial-and-error process. Because the quality of street * Inpatient study, drug of abuse heroin is never certain, the addict’s description of the size of the current habit is of minimal value. The safest guide to Figure 27-11. Characteristics of patients treated under the buprenor- dosage is to monitor the patient’s pulse, respiration, and pupil- phine waiver program. lary size and to use opiate withdrawal scales (such as the COWS). After withdrawal is documented, the patient should receive0 1 to 20 mg of methadone orally. Only if the patient is well known as a heavy user should a dose of 30 mg be started. Methadone admissions to TEDS sites A relatively young patient or a patient who reports a small BUP patient study habit can begin treatment with 10 mg given orally. If vital signs 100% have not stabilized or if withdrawal signs reappear after 2 92% 80% hours, an additional 5 or 10 mg can be given orally. It is rare for the patient to require more than 40 mg during the first 24 60% 56% hours. Successful long-term outpatient maintenance treatment 53% 50% 40% 42% generally uses daily doses in the range of 80 to 120 mg, 35% although lower doses may be adequate to control withdrawal 20% 29% Patients treated 18% symptoms in the inpatient setting. A patient with opioid addic- 0% tion should be maintained on a single daily oral dose that Female White Employed Some post- keeps him or her comfortable and that keeps heart rate and secondary respiratory rate within the normal range. The dose should be education reduced by 5 or 10 mg if the patient appears lethargic. Figure 27-12. Methadone patients and buprenorphine (BUP) patients study sample: demographic differences. (The Treatment Episode Data Inpatient Detoxification Set [TEDS] reports primarily on admissions to facilities receiving public funding. Admissions to private facilities are underrepresented.) If the patient is to be withdrawn from drugs immediately, withdrawal symptoms should first be controlled with metha- done. The methadone dose can then be reduced by 10% to In 2010, an injectable form of naltrexone has been approved 20% each day. If the patient has been maintained on metha- byA. FD Injectable form of naltrexone (Vivitrol) appears to be done in the hospital for 2 or more weeks or if the patient has a safe and effective alternative in increasing patient compli- been on methadone treatment before admission, detoxifica- ance and reduce relapse rates.37 tion should proceed more slowly. The dose can be reduced by 50 to 1 mg/day until 20 mg/day is reached. Further dosage Managing the Opiate-addicted reduction should occur more gradually, particularly if the Hospitalized Patient patient experiences significant cravings or withdrawal symp- toms. Symptoms of chronic pain and anxiety disorders, If a patient is already receiving opiate substitution treatment particularly panic disorder, may also intensify during detoxi- before admission to the hospital, the methadone, LAAM, or fication. Symptomatic treatment should be provided as buprenorphine dose should be confirmed by inpatient staff needed. and should not be changed without consultation with the Clonidine, an alpha2-adrenergic agonist, suppresses the physician responsible for the patient’s outpatient treatment. noradrenergic symptoms of withdrawal and can be used as an Under current FDA regulations, hospital-based physicians may alternative medication for withdrawal. The patient should first prescribe methadone, LAAM, or buprenorphine to any be stabilized on methadone. Clonidine should not be substi- addicted hospitalized person without a specialized treatment tuted for methadone until the methadone dose has been waiver or an opioid treatment program registration, as long as reduced to 20 mg/day. After an initial oral dose of 0.2 mg of the patient was admitted for treatment of a condition other clonidine, patients usually require doses in the range of 0.1 to than opiate dependence. The patient should not be withdrawn 0.2 mg every 4–6 hours. The total dose should not exceed from opiate substitution therapy unless there is full agreement 1.2 mg/day. Patients on clonidine should be monitored among the patient, the hospital-based physician, and the out- closely for side effects (particularly hypotension and seda- patient treatment staff on this course of action. Such detoxifi- tion). Clonidine doses should be withheld for a systolic blood cation is rarely successful, particularly if the patient is under pressure below 90 mm Hg, or a diastolic blood pressure stress from a co-morbid medical or surgical condition. With- below 60 mm Hg. In an inpatient setting, clonidine can be drawal from drugs may complicate the management of the tapered and discontinued over 3–4 days.18,19 A transdermal primary illness. The option of detoxification should not be clonidine patch is often applied on the third day. Supplemen- considered until the patient has fully recovered from the con- tal doses of lorazepam can also be used to moderate dition that required hospitalization. Maintenance treatment withdrawal-related anxiety. Because clonidine does not ade- should be continued until the individual has fully recovered quately suppress the subjective symptoms of withdrawal, as

Downloaded for Rohul Amin ([email protected]) at Uniformed Services Univ of the Health Sciences from ClinicalKey.com by Elsevier on September 29, 2018. For personal use only. No other uses without permission. Copyright ©2018. Elsevier Inc. All rights reserved. 300 PART IX Psychiatric Disorders does methadone and buprenorphine, and is relatively ineffec- when a physician discharges a patient for disciplinary reasons, tive for the treatment of muscle aches and insomnia, it is not medical ethics necessitate that the patient be withdrawn from acceptable for many opioid-dependent patients. methadone before discharge. Buprenorphine has also been effective for the short-term According to federal regulations, hospital-based physicians inpatient detoxification of opiate addicts. Patients can usually can treat patients’ withdrawal symptoms for detoxification be detoxified over 3 to 5 days. After a patient shows signs of purposes with methadone or buprenorphine as long as the opiate withdrawal, he or she can be dosed up to buprenorphine/ opioid-dependence diagnosis is incidental for that inpatient naloxone 8 mg/2 mg on day 1, and increased to 12 mg/3 mg admission. Similarly, if a patient who has been maintained on ony da 2. The dose can be cut to 6 mg/1.5 mg on day 3, with methadone by a methadone program, or buprenorphine by additional 2 mg cuts on each of the following days, if required. an outpatient provider, gets admitted for medical reasons, the Opioid-addicted patients usually report that a buprenorphine hospital-based physicians can keep patients on methadone or detoxification is more comfortable than a detoxification buprenorphine for opioid dependence after the treatment with either methadone or clonidine. Rates for successful com- team confirms the dose with the outside provider or the meth- pletion of opiate detoxification with buprenorphine are adone program.24 almost 65% higher than those reported for clonidine.20 Regardless of the form of opiate substitution therapy Chances for successful treatment of withdrawal are employed, all patients require supplemental counseling and enhanced if the patient is aware of the dose and is able rehabilitation services; these should include educational and to choose a withdrawal schedule within the limits established vocational services as needed. CBT has been shown to be by the physician. By involving the patient in the treatment much more effective than drug counseling alone. Contingency process and by using a flexible withdrawal schedule, the physi- management has also helped to reduce illicit drug use in cian can keep withdrawal symptoms to a tolerable level. patients on maintenance therapy. Rigid adherence to a fixed-dosing schedule is less likely to achieve success, and it may lead to premature termination of Outpatient Detoxification Treatment treatment. Other techniques can be used for rapid inpatient detoxifi- The primary purpose of detoxification treatment is to control cation from opiates, but they require more intensive medical withdrawal symptoms while gradually reducing the dose of management. The patient is first stabilized on clonidine as opiates. According to FDA regulations, maintenance clinics described previously, and 12.5 mg of naltrexone is added may extend outpatient detoxification treatment from 30 to orally. Over the next 3 days, clonidine is gradually reduced, 180 days if a briefer detoxification program is not successful. while naltrexone is gradually increased (up to a single dose of The procedures described in the previous section can be easily 150y mg) b the fourth day. A supplemental is adapted to the outpatient setting. The primary advantage of also given for agitation and insomnia. At the end of 5 days, outpatient detoxification is that a more gradual procedure there are no further withdrawal symptoms and the patient can greatly reduces the severity of withdrawal symptoms. While be discharged directly to an outpatient naltrexone program.19,21 the procedures previously described for detoxification with Serious side effects have been reported with this procedure, clonidine can be used on an outpatient basis, the risks associ- including a 25% incidence of delirium that has necessitated ated with hypotension and sedation have generally limited the termination of treatment.22 A more rapid experimental proto- use of clonidine to inpatient settings. col using high doses of opiate antagonists given under general With the approval of SL buprenorphine for the treatment anesthesia permits completion of detoxification in 48 hours.23 of opiate dependence in office-based settings, clinicians have This approach has not been adequately evaluated in rand- reported success with this drug for outpatient detoxification. omized clinical trials, and it cannot be recommended at this Withdrawal symptoms can be adequately controlled within 48 time. hours using the dosing-induction procedure described for Although techniques that permit a safe, rapid, and medi- inpatient detoxification. It is rare for patients to require more cally effective detoxification from opiates seem highly attrac- than 16 mg/4 mg buprenorphine/naloxone to suppress symp- tive in an era of managed care, clinicians must recognize that toms. Protocols for dose reduction have ranged from 3 to 28 detoxification alone is rarely successful as a treatment for any days, with no general consensus on the optimal duration of addiction. Unless an opiate addict is directly transferred to a treatment. All of these protocols have been reported to produce long-term residential treatment program, relapse rates follow- satisfactory results, either equal to, or superior to, methadone, ing detoxification are extremely high. The resulting costs to and consistently superior to clonidine. While those who are the patient, to society, and to the health care system far out- addicted almost uniformly prefer buprenorphine detoxifica- weighy an saving realized from a rapid “cost-effective” detoxi- tion to either methadone or clonidine, it is important to fication protocol. remember that there are few data to support the long-term Discharge planning should be initiated as quickly as pos- efficacy of any detoxification treatment. For even the most sible after admission. For patients who are not already in motivated patients, 12-month relapse rates generally exceed treatment, several weeks may be required to arrange admission 82%.15 to a drug-free residential program or to an opiate substitution therapy program. Physicians able to provide office-based Pain Management for Patients Receiving buprenorphine treatment can be identified via the “physician Opiate Substitution locator” at the SAMHSA website (http://www.buprenorphine .samhsa.gov) or at the website maintained by the National Determining the appropriate dosage of pain medications for Alliance of Advocates for Buprenorphine Treatment (www a patient receiving opiate substitution therapy is a common .naabt.com). Because a serious medical illness usually causes clinical problem. The analgesic effect of methadone is minimal a patient with opioid addiction to re-examine his or her in maintenance patients, and, at best, lasts only 6 to 8 hours. behavior and possibly to choose rehabilitation, the treating If pain control is required, an addicted person should be given physician should always emphasize the need for long-term standard doses of other narcotics in addition to his or treatment. No individual should be discharged while still her maintenance dose of methadone. Because of cross- receiving methadone unless he or she is returning to a main- tolerance, a patient on maintenance narcotic therapy metabo- tenance program or specifically refuses detoxification. Even lizes other narcotics more rapidly and may therefore require

Downloaded for Rohul Amin ([email protected]) at Uniformed Services Univ of the Health Sciences from ClinicalKey.com by Elsevier on September 29, 2018. For personal use only. No other uses without permission. Copyright ©2018. Elsevier Inc. All rights reserved. Drug Addiction 301 more frequent administration of analgesics than might a non-addicted patient. Pentazocine and other partial opiate BOX 27-2 DSM-5 Diagnostic Criteria: , 27 agonists are contraindicated for such patients. Because of their , or Anxiolytic Withdrawal narcotic antagonist effects, these analgesics produce with- drawal symptoms in opiate addicts. If a patient is maintained A. Cessation of (or reduction in) sedative, hypnotic, or anxiolytic on the buprenorphine/naloxone combination tablet and use that has been prolonged. requires additional narcotic analgesia treatment, supplemen- B. Two (or more) of the following, developing within several tal 2 mg/0.5 mg SL doses of buprenorphine/naloxone every hours to a few days after the cessation of (or reduction in) 4–6 hours can be added to the patient’s daily maintenance sedative, hypnotic , or anxiolytic use described in Criterion A: dose. Similar to methadone, buprenorphine will provide more 1. Autonomic hyperactivity (e.g., sweating or pulse rate effective analgesia if the daily maintenance dose is divided greater than 100 bpm). and administered on a TID schedule. If the patient requires 2. Hand tremor. treatment for severe pain, higher-than-usual narcotic doses 3. Insomnia. may be required to overcome the partial antagonist action of 4. Nausea or vomiting. buprenorphine. As long as the supplemental opiate is added 5. Transient visual, tactile, or auditory hallucinations or following the daily buprenorphine dose, there will be no risk illusions. of precipitated withdrawal. Dispensing the other opiate before 6. Psychomotor agitation giving the daily buprenorphine dose must be avoided since 7. Anxiety. buprenorphine will displace the other opiate at the receptor 8. Grand mal seizures. and will precipitate withdrawal. Pain in buprenorphine- C. The signs or symptoms in Criterion B cause clinically maintained patients can also be managed with non-opiate significant distress or impairment in social, occupational, or analgesics, regional analgesia, or conscious sedation.25 Alter- other important areas of functioning. natively, such patients can be switched to methadone and can D. The signs or symptoms are not attributable to another be managed as described previously. medical condition and are not better explained by another mental disorder, including intoxication or withdrawal from another substance. Overdose Prevention and Reversal Specify if: Opiate overdoses are medical emergencies and they require With perceptual disturbances: This specifier may be noted immediate attention to the maintenance of airway, breathing, when hallucinations with intact reality testing or auditory, and circulation (i.e., ABCs of resuscitation). Opiate-induced visual, or tactile illusions occur in the absence of a delirium. respiratory depression can be treated with 0.4 mg/ml of IV or IM naloxone. This medication can be repeated every 2 minutes, Reprinted with permission from the Diagnostic and statistical manual of as needed (up to a total dose of 2 mg). If the patient does not mental disorders, ed 5, (Copyright 2013). American Psychiatric respond after 20 minutes, he or she should be treated for a Association. combined . Because of the long duration of action of methadone and LAAM, overdoses of these drugs often require an IV naloxone drip. Patterns of Chronic Use versus Abuse As a response to the prescription pain-reliever epidemic, many states have implemented intranasal Narcan overdose- Benzodiazepines can produce dependence, especially when prevention programs. Practitioners caring for those who abuse, used in high doses or for prolonged periods. Up to 45% of or are addicted to, prescription opioids or heroin are advised patients receiving stable, long-term doses show evidence of to educate their patients, as well as family and friends, about physiological withdrawal. Withdrawal symptoms, which are the risk of overdose. Intranasal Narcan can be provided free usually the same in both high-dose and low-dose patients, by state public health agencies or prescribed to such high-risk include anxiety, insomnia, irritability, depression, tremor, individuals as well as to household members. Between 2007 nausea, vomiting, and anorexia. Seizures and psychotic reac- and 2011, intranasal Narcan reversed more than 1,000 opiate tions have also been reported. The more common symptoms overdoses in Massachusetts.26 are similar to those seen during withdrawal from all of the sedative-hypnotic drugs, and they may be difficult to distin- BENZODIAZEPINES guish from the symptoms for which the benzodiazepine was originally prescribed (Box 27-2 contains DSM-IV diagnostic In 2011, DAWN estimated 357,836 ED visits for non-medical criteria; refer to Box 27.3 for DSM-5 criteria). In general, with- use of prescription benzodiazepines.3 Benzodiazepines drawal symptoms abate within 2 weeks. account for 28.7% of all prescription medication-related ED Benzodiazepines (such as diazepam and alprazolam) with visits, the second highest following narcotic pain-relievers in a rapid onset of action seem to be sought out by drug abusers this category. and are generally presumed to have a greater potential for abuse than benzodiazepines with a slower onset of action (e.g., Neurobiology oxazepam). Nonetheless, there is relatively little evidence for the abuse of benzodiazepines when they are prescribed for Benzodiazepines, classified as GABA agonists, bind to a legitimate medical conditions. Ciraulo and collagues27 found subunit of the GABA receptor; of note, GABA is the major that the use patterns of benzodiazepines, even among former inhibitory neurotransmitter in the CNS. Attachment to the alcoholics, were similar to those of other psychiatric patients. receptor opens chloride ion channels and increases the electric A study of alcoholics conducted at the Addiction Research gradient across the cell membrane, thus making the neuron Foundation of Ontario found that 40% were recent users of less excitable. Their primary clinical effects are sedation, a benzodiazepines and that there was a 20% life-time incidence reduction in anxiety, and an increase in the threshold. of anxiolytic abuse or dependence.28 Although these studies Long-term binding to the GABA receptor can alter the number suggest that concerns about the abuse of benzodiazepines by of receptors or change the affinity of the ligand for the alcoholics can be exaggerated, the problem can occur in some receptor. patients. Anxiolytics may be prescribed to this population, but

Downloaded for Rohul Amin ([email protected]) at Uniformed Services Univ of the Health Sciences from ClinicalKey.com by Elsevier on September 29, 2018. For personal use only. No other uses without permission. Copyright ©2018. Elsevier Inc. All rights reserved. 302 PART IX Psychiatric Disorders

be warned about a temporary increase in anxiety symptoms. BOX 27-3 DSM-5 Diagnostic Criteria: Sedative, The simplest approach to detoxification is a gradual reduction Hypnotic, or Anxiolytic Intoxication in dose that may be extended over several weeks or months; under no circumstances should benzodiazepines be stopped A. Recent use of a sedative, hypnotic, or anxiolytic. abruptly. When a more rapid detoxification is desired, inpa- B. Clinically significant maladaptive behavioral or psychological tient dosage reduction can be completed within 2 weeks. For changes (e.g., inappropriate sexual or aggressive behavior, some patients, this rapid withdrawal process produces an mood lability, impaired judgment) that developed during, or unacceptable level of subjective distress. An alternative shortly after, sedative, hypnotic, or anxiolytic use. approach is to switch to a high-potency, long-acting benzodi- C. One (or more) of the following signs or symptoms developing azepine (such as clonazepam). Most patients seem to tolerate during, or shortly after, sedative, hypnotic, or anxiolytic use: detoxification on clonazepam quite well. Because of the pro- 1. Slurred speech longed self-taper after completion of detoxification with clon- 2. Incoordination azepam, patients experience a smoother course of withdrawal 3. Unsteady gait with a minimum of rebound anxiety.31,32 An alternative 4. Nystagmus approach for inpatient detoxification is a 3- to 4-day taper 5. Impairment in cognition (e.g., attention, memory) using anticonvulsants. Carbamazepine and valproate are the 6. Stupor or coma. best-studied medications for this purpose, though topiramate D. The signs or symptoms are not attributable to another and gabapentin are probably effective. medical condition and are not better explained by another Withdrawal from the high-potency, short-acting benzodi- mental disorder, including intoxication with another azepines (such as alprazolam) has been particularly problem- substance. atic. A rapid tapering of these drugs is often poorly tolerated Reprinted with permission from the Diagnostic and statistical manual of by, patients and a switch to equivalent doses of a long-acting mental disorders, ed 5, (Copyright 2013). American Psychiatric benzodiazepine often allows acute withdrawal symptoms to Association. emerge. In general, clonazepam is substituted for alprazolam, at a dose ratio of 0.5 mg of clonazepam for each 1 mg of alprazolam. Clonazepam should then be continued for 1 to 3 weeks. A drug taper is not always required, although abrupt discontinuation of even a long-acting agent (such as clon- they should never be a first-line treatment, and patients must azepam) can be associated with a withdrawal syndrome that always be carefully monitored for signs of abuse. There is no includes seizures, but it tends to occur several days after dis- evidence that anxiolytics are effective as a primary treatment continuation. A 2- to 3-week taper is usually adequate. for or for drug dependence. Supplemental medication is of little use during benzodi- It is important to distinguish between a drug abuser who azepine withdrawal; beta-adrenergic blockers (e.g., pro- uses benzodiazepines primarily to get high, often deliberately pranolol) and alpha-adrenergic agonists (e.g., clonidine) offer mixing them with alcohol and other drugs of abuse, and an no advantage over detoxification using benzodiazepines alone. individual who takes benzodiazepines appropriately under Although they tend to moderate the severity of physiological medical supervision. In both cases, the user may develop symptoms, they are ineffective in controlling the patient’s sub- physiological and psychological dependence. Such depend- jective sense of anxiety, and they do not prevent withdrawal ence, in and of itself, is not evidence of addiction or drug seizures or delirium. Buspirone has no cross-tolerance for the abuse. Unless there is evidence of dose escalation, the deliber- benzodiazepines and does not control withdrawal symptoms ate use to produce a high, or dangerous states of intoxication, from this class of drugs. there is no reason to assume that chronic benzodiazepine users are abusers. Even though clonazepam is the only benzo- diazepine with an indication for long-term use, common SEDATIVE- medical practice supports the merit of the continued use of benzodiazepines in some individuals with chronic medical Abuse and psychiatric conditions. Use of CNS depressants accounts for high rates of ED visits related to suicidal attempts and accidental overdoses (conse- Overdose quent to recreational use and self-medication). Although ben- zodiazepines have become the most commonly abused Flumazenil, a specific benzodiazepine antagonist, reverses the sedative-hypnotics in the US, there are still areas where the life-threatening effects of a . An non-medical use of (such as butalbital [Fiorinal initial IV dose of 0.2 mg should be given over 30 seconds, and Esgic]), carisoprodol (Soma), and other sedative-hypnotics followedy b a second 0.2 mg IV dose if there is no response (such as methaqualone, glutethimide) causes serious clinical after 45 seconds. This procedure can be repeated at 1-minute problems. More recently, a significant increase with zolpidem- intervals (up to a cumulative dose of 5 mg). This treatment is related ED visits has been noted.33 contraindicated in individuals dependent on benzodiazepines or those taking tricyclic antidepressants (TCAs), because flumazenil may precipitate seizures in these patients.29,30 When Clinical Syndromes flumazenil is contraindicated, benzodiazepine overdoses A person intoxicated on a CNS depressant typically has many should be handled similarly to other sedative-hypnotic over- of the same diagnostic features associated with alcohol intoxi- doses (see the following section). cation. Slurred speech, unsteady gait, and sustained vertical or horizontal nystagmus that occur in the absence of the odor of Withdrawal alcohol on the breath suggest the diagnosis. Unfortunately, since drug abusers frequently combine alcohol with other In cases in which there is clear evidence of benzodiazepine sedative-hypnotics, the clinician may be misled by the odor abuse or when the patient desires to stop using these medica- of alcohol. The diagnosis of mixed alcohol– intoxi- tions, it is important that detoxification occurs under medical cation can be missed unless a careful history is taken and supervision. During the withdrawal process, patients should blood and urine samples are analyzed for toxic drugs. The

Downloaded for Rohul Amin ([email protected]) at Uniformed Services Univ of the Health Sciences from ClinicalKey.com by Elsevier on September 29, 2018. For personal use only. No other uses without permission. Copyright ©2018. Elsevier Inc. All rights reserved. Drug Addiction 303 behavioral effects of barbiturate intoxication can vary develop, and dangerous hyperpyrexia can occur in severe widely, even in the same person, and may change significantly cases. Major withdrawal symptoms (such as convulsions and 27 depending on the surroundings and on the expectations of the delirium) indicate addiction to large doses (more than user. Individuals using barbiturates primarily to control 900y mg/da of secobarbital). anxiety or stress may appear sleepy or mildly confused as a Because of the danger of convulsions, barbiturate with- result of an overdose. In young adults seeking to get high, a drawal should be carried out only on an inpatient basis. Grand similar dose may produce excitement, loud boisterous behav- mal seizures, if they occur, are usually seen between the third ior, and loss of inhibitions. The aggressive and even violent and seventh days, although there have been cases reported of behavior commonly associated with may convulsions occurring as late as 14 days after the completion follow. The prescribed regimen for managing an angry alcohol of a medically controlled detoxification. Withdrawal seizures abuser can also be used for the disinhibited abuser of are thought to be related to a rapid drop in the blood barbitu- sedative-hypnotics. rate level. Treatment should therefore be carefully controlled As tolerance to barbiturates develops, there is no concomi- so that barbiturates are withdrawn gradually, with minimal tant increase in the lethal dose, as occurs in opiate depend- fluctuation in the blood barbiturate level. Theoretically, this ence. Although the opiate addict may be able to double his or should decrease the danger of convulsions. Treatment with her regular dose and still avoid fatal respiratory depression, as phenytoin does not prevent convulsions caused by barbiturate little as a 10%–25% increase over the usual daily dosage may withdrawal, although it controls convulsions caused by be fatal to the barbiturate addict. Thus, a epilepsy. should always be considered potentially life-threatening, espe- Withdrawal delirium occurs less frequently than do convul- cially in a drug abuser. sions, and it rarely appears unless preceded by convulsions. It In overdose, a variety of signs and symptoms may be usually begins between the fourth and sixth days after drug observed, depending on the drug or the combination of drugs use is stopped and is characterized by both visual and auditory used, the amount of time since ingestion, and the presence of hallucinations, delusions, and fluctuating level of conscious- complicating medical conditions (e.g., pneumonia, hepatitis, ness. The presence of confusion, hyperreflexia, and fever helps diabetes, heart disease, renal failure, or head injury). Initially distinguish this syndrome from schizophrenia and other non- the patient appears lethargic or semi-comatose. The pulse rate toxic psychoses. is slow, but other vital functions are often normal. As the level of intoxication increases, the patient becomes unresponsive to Treatment for Withdrawal painful stimuli, reflexes disappear, and there is a gradual depression of the respiratory rate; eventually cardiovascular Several techniques are available for the management of bar- collapse ensues. Pupillary size is not changed by barbiturate biturate withdrawal. The basic principle is to withdraw the intoxication, but secondary anoxia may cause fixed, dilated addicting agent slowly to avoid convulsions. First, the daily pupils. In persons who have adequate respiratory function, dosage that produces mild toxicity must be established. pin-point pupils usually indicate an opiate overdose or the Because barbiturate addicts tend to underestimate their drug combined ingestion of barbiturates and opiates. Such patients use, it is dangerous to accept the patient’s history as com- should be observed carefully for increased lethargy and for pletely accurate. Treatment should begin with an oral test dose progressive respiratory depression. Appropriate measures for of 200 mg of pentobarbital, a short-acting barbiturate. If no treating overdoses should be instituted as necessary. Patients physical changes occur after 1 hour, the patient’s habit prob- should not be left unattended until all signs of intoxication ably exceeds 1,200 mg of pentobarbital per day. If the patient have cleared. shows only nystagmus and no other signs of intoxication, the Because there is no cross-tolerance between narcotics and habit is probably about 800 mg/day. Evidence of slurred barbiturates, special problems are presented by patients receiv- speech and intoxication, but not sleep, suggests a habit of 400 ing methadone or buprenorphine maintenance who continue to 600 mg/day. The patient can then be given the estimated to abuse sedative-hypnotics. If a barbiturate overdose is sus- daily requirement divided into four equal doses administered pected, the opiate-dependent patient should be given a nar- orally every 6 hours. Should signs of withdrawal appear, the cotic antagonist to counteract any respiratory depression estimated daily dosage can be increased by 25% following an causedy b the opiate. Naloxone hydrochloride (Narcan) additional dose of 200 mg of pentobarbital given intramuscu- 0.4 mg is given IM or IV because it is a pure narcotic antago- larly. After a daily dose that produces only mild toxicity has nist and it has no respiratory depressant effect, even in large been established, is substituted for pentobarbi- doses. If respiratory depression does not improve after treat- tal0 (3 mg phenobarbital equals 100 mg pentobarbital) and ment with naloxone, the patient should be treated for a pure then withdrawn at a rate of 30 to 60 mg/day34 (Table 27-3). barbiturate overdose. Supportive measures include mainte- nance of adequate airway, mechanic ventilation, alkaliniza- tion of the urine, correction of acid–base disorders, and diuresis with furosemide or mannitol. Severe overdose cases TABLE 27-3 Equivalent Doses of Common Sedative-Hypnotics may require dialysis or charcoal resin hemoperfusion.30 Generic Name Dose (mg)

BARBITURATES Withdrawal Phenobarbital 30 Secobarbital 100 Withdrawal from sedative-hypnotics can present with a wide Pentobarbital 100 variety of signs and symptoms (including anxiety, insomnia, BENZODIAZEPINES hyperreflexia, diaphoresis, nausea, and vomiting), or some- times delirium and convulsions. As a general rule, individuals Alprazolam 1 Diazepam 10 who ingest 600 to 800 mg/day of secobarbital for more than Chlordiazepoxide 25 45 days develop physiological addiction and show symptoms Lorazepam 2 after taper or discontinuation. Minor withdrawal symptoms Clonazepam 0.5–1 usually begin within 24–36 hours after the last dose. OTHERS Pulse and respiration rates are usually elevated, pupil size is Meprobamate 400 normal, and there may be postural hypotension. Fever may

Downloaded for Rohul Amin ([email protected]) at Uniformed Services Univ of the Health Sciences from ClinicalKey.com by Elsevier on September 29, 2018. For personal use only. No other uses without permission. Copyright ©2018. Elsevier Inc. All rights reserved. 304 PART IX Psychiatric Disorders

The long-acting barbiturate phenobarbital is the drug of choice for managing detoxification for drugs in this class. BOX 27-4 Inpatient Management and Referral: Keys to Cross-tolerance with benzodiazepines is incomplete. A dose Successful Inpatient Treatment of0 3 mg of phenobarbital can be substituted for each 100 mg of other barbiturates, or 200 mg of meprobamate or • Perform a psychiatric evaluation 400 mg of carisoprodol (Soma). An alternative method is to • Develop a long-term treatment plan treat emerging withdrawal symptoms orally with 30 to 60 mg • Demonstrate explicit concern and expertise of phenobarbital hourly, as needed, for 2–7 days. After the • Expect testing behavior patient has received similar 24-hour doses for 2 consecutive • Set appropriate limits days, the 24-hour stabilizing dose is given in divided doses • Limit and monitor visitors every 3 to 6 hours. A gradual taper is then instituted as • Supervise passes described previously. This latter method is recommended • Monitor urine for illicit drug use because the use of a long-acting barbiturate produces fewer • Encourage ward activities and recreation variations in the blood barbiturate level and should produce • Initiate family/network therapy a smoother withdrawal. • Treat with respect

Inpatient Management and Referral residential care should be made early in the treatment process. Those addicted to sedative-hypnotic drugs can present with a Ideally, the patient should meet the future therapist before variety of psychological management problems. Effective discharge. Alcoholics Anonymous and Narcotics Anonymous treatment requires a thorough evaluation of the patient’s are useful adjuncts to any outpatient treatment program. If psychiatric problems and the development of long-term treat- transferring to a halfway house or residential program, the ment plans before discharge. Treatment for withdrawal or patient should move there directly from the hospital. Addicts overdose presents an opportunity for effective intervention for are not likely to execute plans for follow-up care without thes addict’ self-destructive lifestyle. Drug abuse patients have strong encouragement and support. a reputation for deceit, manipulation, and hostility. They fre- quently sign out against medical advice. It is rarely acknowl- Bath Salts edged that these problems are sometimes caused by clinicians who fail to give appropriate attention to the patient’s psycho- Since 2010 there has been a rapid increase in the number of logical problems. Most of these difficulties can be eliminated reports of the abuse of various synthetic derivatives of meth- bye effectiv medical and psychiatric management. The patient’s cathinone, either mephedrone or methylenedioxypyrovalerone lack of cooperation and frequent demands for additional (MDPV).35 These compounds are DEA schedule I substances drugs are often the result of anxiety and the fear of withdrawal that are considered illegal only if intended for human con- seizures. This anxiety is greatly relieved if the physician sumption. Typically they have been sold legally and labeled thoroughly explains the withdrawal procedure and assures “plant food or bath salts, not for human consumption,” the patient that the staff members know how to handle though it is apparent that they were never intended for use as withdrawal and that convulsions will be avoided if the bath. salts In 2011, the DEA, using its emergency scheduling patient cooperates with a schedule of medically supervised authority, made sale and possession of these substances illegal withdrawal. in the US. These substances are derived from Khat (or qat; Physicians sometimes fail to realize that the patient’s tough, Catha edulis), a flowering plant from East Africa. They contain demanding behavior is a defense against a strong sense of cathione, an amphetamine analogue and they have been personal inadequacy and a fear of rejection. Addicts have been chewed in some African cultures for centuries. Recently, cathi- conditioned to expect rejection and hostility from medical none has been isolated from the Khat plant and has been personnel. The trust and cooperation necessary for successful transformed into the more potent methcathinone (ephe- treatment cannot be established unless physicians show by drine).36 It is sold as a white or tan crystalline powder that can their behavior that they are both genuinely concerned about be ingested orally, or administered nasally, rectally, IM, or IV. the patient and medically competent to treat withdrawal. Phy- The average dose is 5 to 20 mg; psychoactive effects begin at sicians can expect an initial period of defensive hostility and 3 to 5 mg. The typical package contains 500 mg, thus creating testing behavior and should not take this behavior personally. a high risk for overdose. Consumption increases intracellular Patients need to be reassured that their physician is sincerely dopamine and serotonin by their effects on dopamine and concerned about them. serotonin reuptake transporters. Prolonged exposure leads to If the patient manifests signs of a character disorder and addictive patterns of use. Subjective effects include euphoria, has a history of severe drug abuse, the setting of firm limits is empathic mood, sexual stimulation, greater mental focus, and necessary to ensure successful treatment. Visitors must be enhanced energy (users typically report feelings similar to limited to those individuals of known reliability. This may those produced by MDMA). The peak “rush” occurs at mean excluding spouses and other relatives. Urine should be 90, minutes with effects lasting for 3–4 hours. The total experi- monitored periodically for use of illicit drugs. Family coun- ence lasts for 6–8 hours and it is often followed by a crash. seling should be started during hospitalization and should More intense psychic effects can include panic attacks, agita- focus on the family’s role in helping the patient develop a tion, paranoia, hallucinations, psychosis, and aggressive, successful long-term treatment program. Hospital passes violent, or bizarre self-destructive behavior; use may also lead should not be granted until detoxification is completed; to anorexia, delirium, and depression. Physical effects included however, passes with staff members as escorts should be used tachycardia, hypertension, mydriasis, arrhythmias, hyperther- as much as possible. An active program of recreational and mia, sweating, rhabdomyolysis, seizures, stroke, cerebral physical therapy is necessary to keep young, easily-bored edema, myocardial infarction, cardiovascular collapse, and patients occupied. Keys to successful inpatient treatment are death. The differential diagnosis includes abuse of cocaine, summarized in Box 27-4. amphetamines, LSD or PCP, as well as serotonin syndrome, Because treatment for detoxification or for an overdose neuroleptic malignant syndrome, or anticholinergic toxicity. rarely cures an addict, referrals for long-term outpatient or Bath salts will not be detected by routine toxic screens. Clinical

Downloaded for Rohul Amin ([email protected]) at Uniformed Services Univ of the Health Sciences from ClinicalKey.com by Elsevier on September 29, 2018. For personal use only. No other uses without permission. Copyright ©2018. Elsevier Inc. All rights reserved. Drug Addiction 305

TABLE 27-4 Equivalent Doses of Narcotic Pain Medications 8. American Psychiatric Association. Practice guidelines: treatment of patients with substance use disorders. Am J 27 Generic Name Parenteral Dose (mg) 163(Suppl.):8, 2006. Morphine 10 9. Johnston LD, O’Malley PM, Bachman JG, et al. Monitoring the Oxycodone (Percocet, OxyContin) 5–10 Future national results on drug use: 2012 Overview, Key Findings on Hydrocodone (Vicodin, Lortab) 10 Adolescent Drug Use, Ann Arbor 2013, Institute for Social Research, Meperidine (Demerol) 100 The University of Michigan. Hydromorphone (Dilaudid) 2.5 10. Centers for Disease Control and Prevention. Reported US AIDS Methadone 5 casesy b HlV-exposure category—1994. MMWR 44:4, 1995. Heroin 10 11. Substance Abuse and Mental Health Services Administration, Center for Behavioral Health Statistics and Quality. Drug Abuse Warning Network, 2008: National Estimates of Drug-Related Emer- gency Department Visits, HHS Publication No. SMA 11–4618, Rock- management usually involves benzodiazepines for acute agita- ville, MD, 2011, HHS. tion; antipsychotics should be used with caution because of 12. CDC. Vital Signs: Overdoses of Prescription Opioid Pain the risk of rhabdomyolysis, arrhythmias, seizures, and NMS. Relievers—United States, 1999–2008. MMWR 60:1–6, 2011. Cardiac monitoring, and IV fluids are recommended; auto- 13. American Psychiatric Association. Diagnostic and statistical manual of mental disorders, ed 5, Washington, DC, 2013, American Psychi- nomic instability may require monitoring in an ICU. atric Association. 14. , Ling W Charuvastra C, Collins JF, et al. Buprenorphine mainte- MIXED-DRUG ADDICTION nance treatment of opiate dependence: a multicenter, randomized clinical trial. Addiction 93:475–486, 1998. Increasing numbers of patients are addicted to varying com- 15. Ball JC, Ross A. The effectiveness of methadone maintenance treat- binations of drugs, including benzodiazepines, cocaine, ment, New York, 1991, Springer-Verlag. alcohol, and opiates. Accurate diagnosis is difficult because of 16. US Department of Health and Human Services, Substance Abuse confusing, inconsistent physical findings and unreliable and Mental Health Services Administration, Center for Substance histories. Blood and urine tests for drugs are required to Abuse Treatment. Evaluation of the buprenorphine waiver program, Presented at American Society of . San Diego, confirm the diagnosis. A patient who is addicted to both May 5, 2006. opiates and sedative-hypnotics should be maintained on 17. Fultz JM, Senay EC. Guidelines for the management of hospital- methadone or buprenorphine while the barbiturate or other ized narcotics addicts. Ann Intern Med 82:815–818, 1975. sedative-hypnotic is withdrawn. Then the methadone or 18. Charney DS, Sternberg DE, Kleber HD, et al. The clinical use of buprenorphine can be withdrawn in the usual manner. Dose clonidine in abrupt withdrawal from methadone. Arch Gen Psy- equivalents of narcotics are provided in Table 27-4. chiatry 38:1273–1277, 1981. Behavioral problems should be dealt with as previously 19. Jaffe JH, Kleber HD. Opioids: general issues and detoxification. described. Firm limit-setting is essential to the success of any In American Psychiatric Association: Treatment of psychiatric dis- effective psychological treatment program. Some patients who orders: a task force report of the American Psychiatric Association, vol. 2, Washington, DC, 1989, American Psychiatric Association. overdose or have medical problems secondary to drug abuse 20. Fingerhood MI, Thompson MR, Jasinski DR. A comparison of (such as subacute bacterial endocarditis and hepatitis) are not clonidine and buprenorphine in the outpatient treatment of physiologically addicted to any drug despite a history of opiate withdrawal. Subst Abus 22:193–199, 2001. multiple-drug abuse. Their drug abuse behavior is usually 21. O’Connor PG, Waugh ME, Carrol KM, et al. Primary care-based associated with severe psychopathology. These patients should ambulatory opioid detoxification. J Gen Intern Med 10:255–260, receive a thorough psychiatric evaluation and may require 1995. long-term treatment. 22. Golden SA, Sakhrani DL. Unexpected delirium during rapid opioid detoxification (ROD). Addict Dis 23(1):65–75, 2004. Access a list of MCQs for this chapter at https://expertconsult 23. Legarda JJ, Gossop M. A 24-h inpatient detoxification treatment .inkling.com for heroin addicts: a preliminary investigation. Drug Alcohol Depend 35:91–95, 1994. 24. Code of Federal Regulations, Chapter II, Drug Enforcement Administration, Department of Justice (4-1-04 Edition). REFERENCES § 1306.07 Administering or dispensing of narcotic drugs. 1. O’Brien CP, McLellan AT. Myths about the treatment of addiction. 25. Alford DP, Compton P, Samet JH. Acute pain management for Lancet 347:237–240, 1996. patients receiving maintenance methadone or buprenorphine 2. Substance Abuse and Mental Health Services Administration. therapy. Ann Intern Med 144:127–134, 2006. Results from the 2011 National Survey on Drug Use and Health: 26. Massachusetts Department of Public Health (MDPH). Overdose Summary of National Findings, NSDUH Series H-44, HHS Publica- Education and Naloxone Distribution (OEND) Program Data, tion No. (SMA) 12-4713, Rockville, MD, 2012, Substance Abuse 2011. and Mental Health Services Administration. 27. Ciraulo D, Sands B, Shader R. Critical review of liability for ben- 3. Substance Abuse and Mental Health Services Administration. zodiazepine abuse among alcoholics. Am J Psychiatry 145:1501– Drug Abuse Warning Network, 2011: National Estimates of Drug- 1506, 1988. Related Emergency Department Visits, HHS Publication No. (SMA) 28. Ross HE. Benzodiazepine use and anxiolytic abuse and depend- 13–4760, DAWN Series D-39, Rockville, MD, 2013, Substance ence in treated alcoholics. Addiction 88:209–218, 1993. Abuse and Mental Health Services Administration. 29. Weinbroum A, Halpern P, Geller E. The use of flumazenil in the 4. Boscarino J, Rukstalis M, Hoffman S. Prevalence of prescription management of acute drug poisoning: a review. Intensive Care Med opioid-use disorder among chronic pain patients: Comparison of 17(Suppl. 1):S32–S38, 1991. the DSM-5 vs. DSM-4 diagnostic criteria. J Addict Dis 30(3):185– 30. Wiviott SD, Wiviott-Tishler L, Hyman SE. Sedative-hypnotics and 194, 2011. anxiolytics. In Friedman L, Fleming NF, Roberts DH, et al., editors: 5. Volkow ND, Fowler JS, Wang GJ. The addicted human brain: Source book of substance abuse and addiction, Baltimore 1996, Wil- insights from imaging studies. J Clin Invest 111:1444–1451, 2003. liams & Wilkins. 6. , Gawin F Kleber H. Abstinence symptomatology and psychiatric 31. Herman JB, Rosenbaum JF, Brotman AN. The alprazolam to clon- diagnosis in cocaine abusers. Arch Gen Psychiatry 43:107–113, azepam switch for the treatment of panic disorder. J Clin Psychop- 1986. harmacol 7:175–178, 1987. 7. Carroll KM. Relapse prevention as a psychosocial treatment 32. Patterson JF. Withdrawal from alprazolam dependency using approach: a review of controlled clinical trials. Exp Clin Psychop- clonazepam: clinical observations. J Clin Psychiatry 51:47–49, harmacol 4:46–54, 1996. 1990.

Downloaded for Rohul Amin ([email protected]) at Uniformed Services Univ of the Health Sciences from ClinicalKey.com by Elsevier on September 29, 2018. For personal use only. No other uses without permission. Copyright ©2018. Elsevier Inc. All rights reserved. 306 PART IX Psychiatric Disorders

33. SAMHSA, Center For Behavioral Health Statistics and Quality. The ADM 98–4308), Rockville, MD, 1998, National Institute on Drug Dawn Report. Emergency department visits for adverse reactions Abuse. involving the insomnia medication Zolpidem, Rockville, MD, 2013, Galanter M, Kleber HD, editors: The American Psychiatric Publishing SAMHSA. textbook of substance abuse treatment, ed 3, Washington, DC, 2004, 34. Smith DE, Wesson DR. Phenobarbital technique for treatment of American Psychiatric Publishing, Inc. barbiturate dependence. Arch Gen Psychiatry 24:56–60, 1971. Graham AW, Schultz TK, Mayo-Smith MF, et al., editors: Principles of 35. Ross EA, Watson M, Goldberger B. “Bath salts” intoxication. N addiction medicine, ed 3, Chevy Chase, MD, 2007, American Engl J Med 365(10):967–968, 2011. Society of Addiction Medicine. 36. Coppola M, Mondola R. Synthetic cathinones: Chemistry, phar- Kranzler HR, Ciraulo DA. Clinical manual of addiction psychopharma- macology and toxicology of a new class of designer drugs of abuse cology, Washington, DC, 2005, American Psychiatric Publishing, marketed as ”bath salts” or ”plant food”. Toxicol Lett 211(2):144– Inc. 149, 2012. McNicholas L, Howell EF. Buprenorphine clinical practice guidelines, 37. Syed YY, Keating GM. Extended-release intramuscular naltrexone Rockville, MD, 2000, Substance Abuse Mental Health Services (VIVITROL®): a review of its use in the prevention of relapse to Administration, Center for Substance Abuse Treatment. Available opioid dependence in detoxified patients. CNS Drugs 27(10):851– at: . 861, 2013. Miller WR, Rollnick S. Motivational interviewing: preparing people for change, ed 2, New York, 2002, Guilford Press. SUGGESTED READING Smith DE, Wesson DR. Diagnosis and treatment of adverse reactions to sedative-hypnotics, (DHHS pub no ADM 75–144), Rockville, MD, Carroll KM. A cognitive-behavioral approach: treating cocaine addiction. 1974, National Institute on Drug Abuse. NIDA therapy manuals for drug addiction series (DHHS pub no

Downloaded for Rohul Amin ([email protected]) at Uniformed Services Univ of the Health Sciences from ClinicalKey.com by Elsevier on September 29, 2018. For personal use only. No other uses without permission. Copyright ©2018. Elsevier Inc. All rights reserved. Drug Addiction 306.e1

MULTIPLE CHOICE QUESTIONS Q6 Which of the following is generally considered to be an alpha2-adrenergic agonist? 27 Select the appropriate answer. ○ Amitriptyline Q1 The signs and symptoms of acute cocaine intoxication ○ Buprenorphine are MOST similar to those of intoxication with which of the following agents? ○ Clonazepam ○ Alprazolam ○ Clonidine ○ Amphetamine ○ Dextroamphetamine

○ Carbon monoxide Q7 Which of the following drugs is MOST likely to ○ Haloperidol counteract the effects of a benzodiazepine overdose? ○ Phenobarbital ○ Benztropine ○ Flumazenil Q2 Withdrawal reactions (manifest by hypertension, ○ Haloperidol tachycardia, hyperreflexia, and fever) are LEAST likely when abruptly discontinuing after chronic use from ○ Naloxone which of the following drugs? ○ Physostigmine ○ Alcohol ○ Alprazolam MULTIPLE CHOICE ANSWERS ○ Cocaine Q1 The answer is: Amphetamine. ○ Secobarbital The signs and the symptoms of acute cocaine intoxication are ○ Lorazepam similar to those of amphetamine abuse. Typical complaints associated with intoxication include anorexia, insomnia, anxiety, hyperactivity, and rapid speech and thought processes Q3 Long-term use of which of the following agents is (“speeding”). Signs of adrenergic hyperactivity (such as hyper- MOST closely linked with a paranoid psychosis? reflexia, tachycardia, diaphoresis, and dilated pupils respon- ○ Alcohol sive to light) may also be seen. More severe symptoms (such as hyperpyrexia, hypertension, and cocaine-induced vasospas- ○ Alprazolam tic events [e.g., stroke or myocardial infarction]) are relatively ○ Amitriptyline rare among users, but are fairly common in those seen in hospital emergency departments. Patients may also manifest ○ Dextroamphetamine stereotyped movements of the mouth, face, or extremities. ○ Lorazepam The most serious psychiatric problem associated with chronic cocaine use is a cocaine-induced psychosis (manifest by visual Q4 The cluster of signs and symptoms (i.e., dysphoric and auditory hallucinations and paranoid delusions often mood, nausea, vomiting, body aches, lacrimation, associated with violent behavior). Tactile hallucinations rhinorrhea, pupillary dilation, sweating, piloerection, (called “coke bugs”) involve the perception that something is diarrhea, yawning, fever, insomnia, irritability, and drug crawling under the skin. A cocaine psychosis may be indistin- craving) is MOST closely linked with withdrawal from guishable from an amphetamine psychosis, but it is usually which of the following class of drugs? shorter in duration. High doses of stimulants can also cause a state of excitation and mental confusion Drugs (e.g., alpra- ○ Anticholinergics zolam and phenobarbital) that work at the alcohol-benzodi- ○ Antipsychotics azepine-barbiturate receptor tend to cause sedation, not hyperactivity; carbon monoxide in excess leads to somnolence ○ Benzodiazepines and to coma, while haloperidol generally results in behavioral ○ Opiates calm, not adrenergic excess. ○ Tricyclic antidepressants Q2 The answer is: Cocaine. Chronic users of cocaine typically follow a cyclical pattern of Q5 Which of the following drugs has been approved for 2 or 3 days of heavy binge use, followed by a withdrawal use in the office-based treatment of opiate dependence “crash.” Detoxification is accomplished by the abrupt cessa- and provides an attractive alternative to methadone tion of all cocaine use, usually through restricted access (e.g., treatment for higher-functioning individuals and for a loss of funds or contacts, or incarceration). The major com- those with shorter histories of opiate dependence? plication of withdrawal is a severe depression with suicidal ideation. A less severe anhedonic state may persist for 2 to 3 ○ Alprazolam months and is thought to reflect a more persistent state of ○ Amitriptyline dopamine depletion. ○ Buprenorphine By contrast, withdrawal from drugs that work at the alcohol- benzodiazepine-barbiturate receptor are closely linked with ○ Dextroamphetamine physiological symptoms (manifest by hypertension, tachycar- ○ Duloxetine dia, hyperreflexia, and fever).

Downloaded for Rohul Amin ([email protected]) at Uniformed Services Univ of the Health Sciences from ClinicalKey.com by Elsevier on September 29, 2018. For personal use only. No other uses without permission. Copyright ©2018. Elsevier Inc. All rights reserved. 306.e2 PART IX Psychiatric Disorders

Q3 The answer is: Dextroamphetamine. should be instructed to refrain from the use of heroin, or any other opiate, for at least 24 hours. Once opiate withdrawal is The signs and symptoms of acute amphetamine intoxication documented (and monitored with an opiate withdrawal scale, are similar to those of cocaine abuse. Long-term effects also such as the Clinical Opiate Withdrawal Scale [COWS]), treat- include depression, brain dysfunction, and weight loss. The ment should begin with 4 mg/1 mg of SL buprenorphine/ other classic syndrome seen in either acute or chronic amphet- naloxone. The patient should be observed for 1 to 4 hours amine intoxication is a paranoid psychosis without delirium. after the initial dose for any signs of precipitated withdrawal. Although typically seen in young people who use intravenous Additional doses of 4 mg/1 mg can be given every 2 to 4 hours methamphetamine hydrochloride, it can also occur in chronic as needed to stabilize the patient. Most clinicians do not pre- users of dextroamphetamine or other amphetamines. scribe more than 12 mg/3 mg on the first day. Should precipi- A paranoid psychosis may also occur with or without other tated withdrawal occur, more aggressive dosing is recommended manifestations of amphetamine intoxication. The absence of to manage the withdrawal symptoms. Most patients can be disorientation distinguishes this condition from most other maintained on SL doses in the range of 12 to 16 mg/day; an toxic psychoses. This syndrome is clinically indistinguishable adequate stabilizing dose can usually be achieved within 2 to from an acute schizophrenic episode of the paranoid type, and 3 days. the correct diagnosis is often made in retrospect, based on a history of amphetamine use and with a urine test that is posi- Q6 The answer is: Clonidine. tive for amphetamines. Use of haloperidol or low-dose atypi- Clonidine, an alpha -adrenergic agonist, suppresses the cal antipsychotics is often effective in the acute management 2 noradrenergic symptoms of withdrawal and can be used as an of this type of substance-induced psychosis. alternative medication for withdrawal. Clonidine should not Other distinctive features of chronic stimulant abuse include be substituted for methadone until the methadone dose has dental problems (e.g., caries, missing teeth, and bleeding and been reduced to 20 mg/day. After an initial oral dose of 0.2 mg infected gums), muscle cramps (related to dehydration and of clonidine, patients usually require doses in the range of 0.1 low levels of magnesium and potassium), constipation (due to 0.2 mg every 4 to 6 hours. The total dose should not exceed to dehydration), nasal perforations, and excoriated skin 1.2 mg/day. Patients on clonidine should be monitored lesions (speed bumps). The urine may have a stale smell due closely for side effects (particularly hypotension and seda- to ammonia constituents used in the illicit manufacture of tion). Clonidine doses should be withheld for a systolic blood methamphetamine. pressure below 90 mm Hg, or diastolic blood pressure below 60 mm Hg. In an inpatient setting, clonidine can be tapered Q4 The answer is: Opiates. and discontinued over 3 to 4 days. A transdermal clonidine patch is often applied on the third day. Because clonidine does The classic signs of opiate withdrawal are easily recognized not adequately suppress the subjective symptoms of with- and usually begin 8 to 12 hours after the last dose (of a short- drawal, as does methadone and buprenorphine, and is rela- lasting agent). The patient generally admits to the need for tively ineffective for the treatment of muscle aches and drugs and shows sweating, yawning, lacrimation, tremor, rhi- insomnia, it is not acceptable for many addicts. norrhea, marked irritability, dilated pupils, piloerection (“gooseflesh”), and an increased respiratory rate. More severe Q7 The answer is: Flumazenil. signs of withdrawal occur 24 to 36 hours after the last dose and include tachycardia, hypertension, insomnia, nausea, Flumazenil, a specific benzodiazepine antagonist, reverses the vomiting, and abdominal cramps. Untreated, the syndrome life-threatening effects of a benzodiazepine overdose. An subsides in 3 to 7 days. Withdrawal symptoms are similar in initial IV dose of 0.2 mg should be given over 30 seconds, patients addicted to methadone, but they may not appear followed by a second 0.2 mg IV dose if there is no response until 24 to 36 hours after the last dose (because of metha- after 45 seconds. This procedure can be repeated at 1-minute done’s longer half-life) and abate over 2 to 4 weeks. Patients intervals (up to a cumulative dose of 5 mg). This treatment is addicted to oxycodone may present a particularly severe and contraindicated in individuals dependent on benzodiazepines prolonged withdrawal syndrome and may require high doses or those taking tricyclic antidepressants (TCAs) because fluma- of opiates for adequate control. zenil may precipitate seizures in these patients. When fluma- zenil is contraindicated, benzodiazepine overdoses should be Q5 The answer is: Buprenorphine. handled similarly to other sedative-hypnotic overdoses. Buprenorphine has been approved for use in the office-based Benztropine may reverse extrapyramidal side effects of treatment of opiate dependence and provides an attractive dopamine blockers, naloxone may reverse opiate intoxication, alternative to methadone treatment for higher-functioning physostigmine may reverse anticholinergic excess, and individuals and for those with shorter histories of opiate haloperidol may reduce the manifestations of psychosis. dependence. To initiate buprenorphine treatment, a patient

Downloaded for Rohul Amin ([email protected]) at Uniformed Services Univ of the Health Sciences from ClinicalKey.com by Elsevier on September 29, 2018. For personal use only. No other uses without permission. Copyright ©2018. Elsevier Inc. All rights reserved.